Laplace tranform of unitstep (heaviside) function

Click For Summary
The discussion revolves around finding the Laplace transform of the function sin(t)[u(t)-u(t-pi)]. The transformation process begins with the expression sin(t)u(t) - sin(t)u(t-pi), but complications arise due to the sine function's argument. Participants express confusion about manipulating the sine term, particularly when it involves the shift by pi. Additionally, a related problem involving e^(-t)u(t-2) is mentioned, highlighting the challenge of dealing with exponentials in Laplace transforms. Clarification on these transformations is sought to facilitate easier computation.
dfang
Messages
1
Reaction score
0

Homework Statement


[/B]
I know for t[u(t)-u(t-2)], we can simplify that to tu(t)-((t-2)+2)u(t-2) which then gives us tu(t)-(t-2)u(t-2)-u(t-2). Now, the laplace transform seems trivial but I am having problems with this equation:
sin(t)[u(t)-u(t-pi)]

Homework Equations


[/B]n/a

3. The Attempt at a Solution

sin(t)[u(t)-u(t-pi)]
=sin(t)u(t) - sin(t)u(t-pi)
= sin(t)u(t) - sin ((t-pi)+pi)u(t-pi)
Where do we go from here as the t-pi+pi is inside the sine i.e its a function of sine. We can't exactly take it out and multiply over. How can we take this to the form where I can take the laplace transform easily ?
Similar problem where I am having trouble: e-t u(t-2). This one I do not even know where to start as the -t is in the exponent this time. Thanks.
 
Last edited by a moderator:
Physics news on Phys.org
Try using this: $$\mathcal{L}f(t)u(t-a)=\int_a^\infty e^{-st}f(t)~dt=\int_0^\infty e^{-s(v+a)}f(v+a)~dv$$ $$ =
e^{-as}\int_0^\infty e^{-sv}f(v+a)~dv =e^{-as}\int_0^\infty e^{-st}f(t+a)~dt=e^{-as}\mathcal{L}f(t+a) $$
 
Last edited:
Question: A clock's minute hand has length 4 and its hour hand has length 3. What is the distance between the tips at the moment when it is increasing most rapidly?(Putnam Exam Question) Answer: Making assumption that both the hands moves at constant angular velocities, the answer is ## \sqrt{7} .## But don't you think this assumption is somewhat doubtful and wrong?

Similar threads

  • · Replies 1 ·
Replies
1
Views
964
  • · Replies 11 ·
Replies
11
Views
3K
  • · Replies 3 ·
Replies
3
Views
2K
  • · Replies 1 ·
Replies
1
Views
2K
Replies
8
Views
1K
Replies
2
Views
2K
  • · Replies 1 ·
Replies
1
Views
2K
Replies
3
Views
2K
  • · Replies 1 ·
Replies
1
Views
2K
  • · Replies 10 ·
Replies
10
Views
2K